Course Content
All Previous Years Krok 2 Papers with Explanations
About Lesson


Question From ( 51 To 100 )


51. A 22 year old woman complained of right subcostal aching pain, nausea, and decreased appetite. She fell ill 2 months after appendectomy when jaundice appeared. She was treated in an infectious hospital. 1 year later above mentioned symptoms developed. On exam: the subicteric sclerae, enlarged fi- rm liver. Your preliminary diagnosis:

A. Chronic viral hepatitis

B. Calculous cholecystitis

C. Gilbert’s disease

D. Acute viral hepatitis

E. Chronic cholangitis


Answer: Chronic viral hepatitis

Explanation

The patient’s symptoms and medical history suggest a diagnosis of chronic viral hepatitis. The right subcostal aching pain, nausea, decreased appetite, and jaundice are all common symptoms of hepatitis. In addition, the fact that the patient had previously been treated for jaundice in an infectious hospital further supports the diagnosis of viral hepatitis.  

Enlargement of the liver is also a common feature of hepatitis, and the firm consistency of the liver suggests that it has become fibrotic due to chronic inflammation.   Calculous cholecystitis is a condition in which gallstones cause inflammation of the gallbladder, leading to abdominal pain and other symptoms.


However, the patient’s symptoms are more consistent with hepatitis than with cholecystitis.   Gilbert’s disease is a benign genetic condition in which the liver is unable to process bilirubin efficiently, leading to mild jaundice. However, the patient’s symptoms are more severe than would be expected with Gilbert’s disease.  

Acute viral hepatitis is a possibility, but the fact that the patient’s symptoms have persisted for over a year suggests a chronic form of the disease.   Chronic cholangitis is a condition in which there is inflammation and scarring of the bile ducts, leading to liver damage. However, the patient’s symptoms are more consistent with chronic viral hepatitis than with cholangitis.


52. A 25 year old woman complained of edema on her face and legs, rise of blood pressure up to 160/100 mm Hg and weakness. She fell ill 3 weeks after recovering from angina. Urinalysis data: protein of 0,5 g/l, erythrocytes of 17-20/field, leukocytes of 2-3/field, erythrocyte casts. What treatment should be initiated after specifying the diagnosis?

A. Penicillin OS

B. Heparin

C. Ceftriaxone

D. Dipyridamole

E. Ciprofloxacine


Answer:  Penicillin OS

Explanation

The patient’s symptoms and urinalysis data suggest a diagnosis of acute glomerulonephritis, which is a type of kidney disease that occurs as a result of inflammation of the glomeruli (the tiny blood vessels in the kidneys that filter waste products from the blood).

The edema, high blood pressure, and presence of protein and red blood cells in the urine (along with erythrocyte casts) are all characteristic of this condition.   Penicillin is not the appropriate treatment for acute glomerulonephritis.

It is important to identify and treat the underlying cause of the condition, which may include a recent streptococcal infection, as well as manage the symptoms. Treatment may involve the use of diuretics to help reduce edema and control blood pressure, as well as medications to reduce inflammation and prevent further damage to the kidneys.  

It is important to consult a healthcare professional for proper diagnosis and treatment of acute glomerulonephritis, as the condition can have serious consequences if left untreated.


53. A fitter of a metallurgic factory with occupational exposure to high concentrations of mercury fumes for 16 years presents instability of pulse and blood pressure, general hyperhydrosis, asymmetric innervations of facial muscles and tongue, positive subcortical reflexes, hand tremor on physical examination. A dentist revealed paradontosis and chronic stomatitis. What is the most probable diagnosis?

A. Chronic mercury intoxication

B. Neuroinfection

C. Parkinson syndrome

D. Acute mercury intoxication

E. Mercury encephalopathy


Answer: Chronic mercury intoxication

Explanation

The patient’s occupation and symptoms suggest a diagnosis of chronic mercury intoxication. Exposure to high concentrations of mercury fumes can lead to a wide range of neurological and other symptoms, including instability of pulse and blood pressure, hyperhidrosis, facial muscle and tongue asymmetry, positive subcortical reflexes, hand tremor, and other neurological abnormalities.  

In addition, the presence of paradontosis and chronic stomatitis may be related to the patient’s occupational exposure, as mercury is a known toxin that can affect the oral cavity and cause dental problems.   Neuroinfection and acute mercury intoxication are less likely diagnoses, as the patient’s symptoms are chronic in nature and have developed over a period of 16 years.

Parkinson syndrome is also a less likely diagnosis, as the patient’s symptoms are not specific to Parkinson’s disease and are more consistent with mercury toxicity.   Mercury encephalopathy is a possible diagnosis, as it can cause a range of neurological symptoms. However, chronic mercury intoxication is a more likely diagnosis given the patient’s occupational history and symptoms.


54. A 42 year old woman complains of dyspnea, edema of the legs and tachycardia during minor physical exertion. Heart borders are displaced to the left and S1 is accentuated, there is diastolic murmur on apex. The liver is enlarged by 5 cm. What is the cause of heart failure?

A. Mitral stenosis

B. Mitral regurgitation

C. Tricuspid stenosis

D. Tricuspid regurgitation

E. Aortic stenosis


Answer: Mitral stenosis

Explanation

The patient’s symptoms and physical examination findings suggest a diagnosis of mitral stenosis, which is a condition in which the opening of the mitral valve (which separates the left atrium and left ventricle of the heart) is narrowed, leading to impaired blood flow and increased pressure in the left atrium.  

The dyspnea, edema of the legs, and tachycardia during minor physical exertion are all symptoms of heart failure, which can occur as a result of mitral stenosis. The displaced heart borders to the left, accentuated S1, and diastolic murmur on apex are all characteristic of mitral stenosis, as is the enlarged liver (which may be due to congestion in the liver as a result of impaired blood flow).  

Mitral regurgitation is a condition in which blood flows back from the left ventricle to the left atrium due to a leaky mitral valve, but this is not consistent with the diastolic murmur heard on apex in this patient, which suggests a narrowed opening.  

Tricuspid stenosis and tricuspid regurgitation involve the tricuspid valve, which separates the right atrium and right ventricle of the heart, and would not lead to displacement of heart borders to the left and an accentuated S1.   Aortic stenosis involves the aortic valve, which separates the left ventricle and aorta, and would not lead to displacement of heart borders to the left, an accentuated S1, or a diastolic murmur on apex.


55. A 32 year old welder complains of weakness and fever. His illness started as tonsillitis a month before. On exam, BT of 38, 9oC, RR of 24/min, HR of 100/min, BP of 100/70 mm Hg, hemorrhages on the legs, enlargement of the lymph nodes. CBC shows Hb of 70 g/l, RBC of 2, 2· 1012/l, WBC of 3, 0· 109/l with 32% of blasts, 1% of eosinophiles, 3% of bands, 36% of segments, 20% of lymphocytes, and 8% of monocytes, ESR of 47 mm/h. What is the cause of anemia?

A. Acute leukemia

B. Chronic lympholeukemia

C. Aplastic anema

D. Vitamin B12 deficiency anemia

E. Chronic hemolytic anemia


Answer: Acute leukemia

Explanation

The patient’s symptoms and laboratory findings suggest a diagnosis of acute leukemia, which is a type of cancer that affects the blood-forming tissues of the body, such as the bone marrow and lymphatic system.   The weakness, fever, enlarged lymph nodes, and presence of blasts (immature white blood cells) in the peripheral blood are all characteristic of acute leukemia.

In addition, the anemia (low hemoglobin and red blood cell count) is a common feature of leukemia, as the abnormal white blood cells can crowd out the normal blood cells in the bone marrow and interfere with their production.  

Chronic lymphocytic leukemia is a type of leukemia that typically affects older individuals and is characterized by the accumulation of mature lymphocytes in the blood and bone marrow. Aplastic anemia is a condition in which the bone marrow fails to produce enough blood cells, leading to anemia and other symptoms.

Vitamin B12 deficiency anemia is a type of anemia caused by a deficiency of vitamin B12, and chronic hemolytic anemia is a type of anemia in which red blood cells are destroyed faster than they can be produced. None of these conditions are consistent with the patient’s symptoms and laboratory findings, which suggest a diagnosis of acute leukemia.


56. A male patient, 60 years old, tobacco smoker for 30 years, alcoholic, has dysphagia and weight loss since 4 months. Suggested diagnosis?

A. Cancer of the esophagus

B. Esophageal achalasia

C. Hanter’s disease

D. Esophagitis

E. Esophageal diverticulum


Answer:  Cancer of the esophagus

Explanation

The patient’s symptoms, age, and risk factors (tobacco smoking and alcohol consumption) suggest a diagnosis of cancer of the esophagus. Dysphagia (difficulty swallowing) and weight loss are common symptoms of esophageal cancer, particularly in the later stages of the disease.  

Esophageal achalasia is a condition in which the lower esophageal sphincter (the muscle at the bottom of the esophagus that controls the passage of food into the stomach) fails to relax properly, leading to difficulty swallowing and other symptoms.

However, the patient’s symptoms are more consistent with esophageal cancer than with achalasia.   Hunter’s disease (also known as mucopolysaccharidosis type II) is a rare genetic disorder that affects the metabolism of certain sugars and can lead to a wide range of symptoms, including skeletal abnormalities, developmental delays, and neurological problems. However, the patient’s symptoms are not consistent with this condition.  

Esophagitis is a condition in which the lining of the esophagus becomes inflamed, leading to symptoms such as pain and difficulty swallowing. However, the patient’s symptoms and risk factors suggest a more serious condition such as cancer.   Esophageal diverticulum is a condition in which a pouch or pocket forms in the wall of the esophagus, which can lead to difficulty swallowing and other symptoms. However, the patient’s symptoms and risk factors suggest a more serious condition such as cancer.


57. Which of the following symptoms would occur only if a total-body acute radiation exposure exceeded 5.000 rad (50 Gy)

A. Hallucinations and impairment of vision

B. Nausea and vomiting

C. Diarrhea

D. Bleeding gums

E. Epilation (hair loss)


Answer: Hallucinations and impairment of vision

Explanation

A total-body acute radiation exposure of 5,000 rad (50 Gy) or more is a very high dose that can lead to a range of severe symptoms, including damage to the central nervous system. Hallucinations and impairment of vision are symptoms that may occur if the central nervous system is affected by radiation exposure at this level.  

Nausea and vomiting are common symptoms of radiation sickness and may occur at lower doses of radiation exposure.

Diarrhea and bleeding gums are also symptoms that may occur at lower doses of radiation exposure.   Epilation (hair loss) is a common symptom of radiation exposure, but it typically occurs at doses of radiation much lower than 5,000 rad (50 Gy). Hair loss may occur at doses as low as 200 rad (2 Gy) or less, depending on the length of exposure and other factors.


58. In the 43rd week of gestation a long, thin infant was delivered. He is apneic, limp, pale, and covered with “pea soup”amniotic fluid. The first step in the resuscitation of this infant at delivery should be:

A. Suction of the trachea under direct vision

B. Artificial ventilation with bag and mask

C. Artificial ventilation with endotracheal tube

D. Administration of 100% oxygen by mask

E. Catheterization of the umbilical vein


Answer: Suction of the trachea under direct vision

Explanation

The infant’s presentation is consistent with meconium aspiration syndrome, which occurs when a fetus passes meconium (the first stool) into the amniotic fluid and subsequently inhales it into the lungs. This can lead to respiratory distress and compromise the infant’s ability to breathe.  

The first step in resuscitating an infant with suspected meconium aspiration syndrome is to suction the trachea under direct vision to remove any meconium that may be obstructing the airway. This can be done using a laryngoscope and suction catheter.  

Artificial ventilation with bag and mask or endotracheal tube may also be necessary if the infant is not breathing adequately after suctioning. Administration of 100% oxygen by mask may be necessary to improve oxygenation, but it should be done after the airway has been cleared and ventilation has been established.  

Catheterization of the umbilical vein may be necessary in certain situations, such as when the infant is in shock and requires immediate access to the bloodstream for medication or fluid resuscitation. However, it is not the first step in the resuscitation of an infant with meconium aspiration syndrome.


59. 25 children at the age of 2-3 years who don’t attend any child welfare institutions should be observed by a district pediatrician within the current year. How many initial visits of this group of children should be planned?

A. 50

B. 20

C. 40

D. 100

E. 200


Answer:  50

Explanation

Since there are 25 children that need to be observed and each child needs an initial visit, the total number of initial visits required is 25.   Therefore, the answer is A. 50.


60. A 25 year old patient complains of pain in the I finger on his right hand. On examination: the finger is homogeneously hydropic, in bent position. On attempt to unbend the finger the pain is getting worse. Acute pain appears during the probe in ligament projection. What decease is the most likely?

A. Thecal whitlow (ligament panaritium)

B. Subcutaneous panaritium

C. Articular (joint) panaritium

D. Bone panaritium

E. Paronychia


Answer: Thecal whitlow (ligament panaritium)

Explanation

The patient’s symptoms and examination findings suggest a diagnosis of thecal whitlow, also known as ligament panaritium or deep space infection. This is an infection of the deep tissues of the finger, including the tendons and ligaments, which can cause pain, swelling, and stiffness.  

The homogeneous hydrops (swelling) of the finger, along with pain during attempts to straighten the finger and during probing in ligament projection, are all characteristic of thecal whitlow. In addition, the patient’s young age and lack of other symptoms suggest a localized infection rather than a systemic illness.  

Subcutaneous panaritium is a superficial infection of the skin and soft tissues around the nail bed, while articular (joint) panaritium involves the joint itself and can cause severe pain and swelling.

Bone panaritium is a rare but serious complication of finger infections in which the infection spreads to the underlying bone. Paronychia is an infection of the nail fold and is typically less severe than the other types of panaritium mentioned.   Therefore, the most likely diagnosis in this case is thecal whitlow (ligament panaritium).


61. A 26 year old manual worker complained of 3 weeks history of fevers and fatigue, weight loss with no other symptoms. Physical findings: Temperature 37, 6oC, Ps- 88 bpm, blood pressure 115/70 mm Hg, superficial lymph nodes (occipital, submental, cervical, axillary) are enlarged, neither tender nor painful. Rubella-like rash on the trunk and extremities. Herpes simplex lesions on the lips. Candidosis of oral cavity. What infectious disease would you suspect?

A. HIV infection

B. Influenza

C. Rubella

D. Infectious mononucleosis

E. Tuberculosis


Answer: HIV infection

Explanation

The patient’s symptoms and physical findings suggest a diagnosis of HIV infection, which is a viral infection that can cause a wide range of symptoms and can affect multiple organ systems.   The patient’s history of fevers, fatigue, weight loss, and lymphadenopathy (enlarged lymph nodes) are all common symptoms of HIV infection, particularly in the early stages of the disease.

The rubella-like rash, herpes simplex lesions, and candidiasis of the oral cavity are also consistent with HIV infection, as these conditions are more common in individuals with weakened immune systems.   Influenza typically presents with more acute onset of symptoms, such as high fever, cough, and muscle aches.

Rubella typically presents with a rash that starts on the face and spreads to the trunk and extremities, but the lymphadenopathy in this case is more widespread and the patient’s other symptoms are not typical of rubella. Infectious mononucleosis can cause similar symptoms to HIV infection, but the presence of candidiasis and herpes simplex lesions suggest a more severe immunodeficiency.

Tuberculosis can cause lymphadenopathy and weight loss, but the other symptoms and physical findings in this case are not typical of tuberculosis.   Therefore, based on the patient’s symptoms and physical findings, HIV infection is the most likely diagnosis.


62. A patient complains about strong dyspnea that is getting worse during physical activity. Presentations appeared suddenly 2 hours ago at work: acute chest pain on the left, cough. The pain was abating, but dyspnea, dizziness, pallor, cold sweat and cyanosis were progressing. Vesicular respiration is absent, X-ray picture shows a shadow on the left. What pathology might be suspected?

A. Spontaneous left-sided pneumothorax

B. Pulmonary infarction

C. Pleuritis

D. Left-sided pneumonia

E. Pulmonary abscess


Answer: Spontaneous left-sided pneumothorax

Explanation

The patient’s symptoms and physical findings suggest a diagnosis of spontaneous left-sided pneumothorax, which is a condition in which air leaks into the space between the lung and the chest wall, causing the lung to collapse.  

The sudden onset of dyspnea, acute chest pain on the left, and cough are all common symptoms of pneumothorax. The progression of dyspnea, dizziness, pallor, cold sweat, and cyanosis suggest that the condition is worsening and compromising the patient’s ability to breathe. The absence of vesicular respiration on the left side and the shadow on the X-ray confirm the diagnosis of pneumothorax.  

Pulmonary infarction, or a blockage of blood flow to the lungs, can also cause sudden onset of dyspnea and chest pain, but it typically presents with symptoms of deep vein thrombosis or pulmonary embolism, such as leg swelling or shortness of breath at rest. Pleuritis, or inflammation of the lining of the lungs, can cause chest pain and dyspnea, but it typically does not cause the sudden onset of symptoms described in this case.

Left-sided pneumonia and pulmonary abscess can also cause dyspnea and cough, but they typically present with fever and localized symptoms such as chest pain or coughing up purulent sputum.   Therefore, based on the patient’s symptoms and physical findings, spontaneous left-sided pneumothorax is the most likely diagnosis.


63. Which of the methods of examination is the most informative in the diagnostics of a tube infertility?

A. Laparoscopy with chromosalpingoscopy

B. Pertubation

C. Hysterosalpingography

D. Transvaginal echography

E. Bicontrast pelviography


Answer: Laparoscopy with chromosalpingoscopy

Explanation

Tube infertility refers to infertility caused by obstruction or damage to the fallopian tubes, which can prevent the egg from reaching the sperm or the fertilized egg from reaching the uterus. There are several methods of examination that can be used to diagnose tube infertility, but laparoscopy with chromosalpingoscopy is considered the most informative.  

Laparoscopy is a minimally invasive surgical procedure in which a small incision is made in the abdomen and a laparoscope (a thin, flexible tube with a camera) is inserted to visualize the pelvic organs. Chromosalpingoscopy involves injecting a dye into the uterus and fallopian tubes and using a laparoscope to observe the flow of the dye through the tubes.

This can help identify any blockages or other abnormalities in the tubes.   Pertubation is a method in which a catheter is inserted into the cervix and dye is injected into the uterus to observe the flow of the dye through the tubes.

Hysterosalpingography is a method in which dye is injected into the uterus through the cervix and X-rays are taken to visualize the flow of the dye through the tubes. Transvaginal echography is an imaging method that uses high-frequency sound waves to visualize the pelvic organs.

Bicontrast pelviography is a radiographic method that uses two contrasting agents to visualize the pelvic organs.   While these methods can also be used to diagnose tube infertility, laparoscopy with chromosalpingoscopy is considered the most informative because it allows for direct visualization of the tubes and any abnormalities. It also allows for the possibility of surgical intervention to correct any blockages or other issues that are identified during the procedure.


64. A 38 year old man worked at roofing and drain pipes production for 15 years. He seeks medical help for expiratory breathlessness on exertion, and dry cough. On exam, wheezes above both lungs, grayish warts on fingers are seen. Factory physician has diagnosed asbestosis. What method is the most important for this diagnosis?

A. Chest X-ray

B. Bronchoscopy

C. Blood gas analysis

D. Spirography

E. Electrocardiography


Answer: Chest X-ray

Explanation

Asbestosis is a lung disease caused by prolonged exposure to asbestos fibers, which can lead to scarring and inflammation of the lung tissue. The most important method for diagnosing asbestosis is chest X-ray, which can show characteristic changes in the lung tissue such as small irregular opacities, diffuse interstitial fibrosis, and pleural thickening.  

In this case, the patient’s history of working in roofing and drain pipes production for 15 years, along with his symptoms of expiratory breathlessness on exertion and dry cough, are consistent with asbestosis. The wheezes above both lungs may be indicative of obstructive airway disease, which can also be caused by exposure to asbestos fibers.

The grayish warts on the fingers, known as “asbestos warts,” are a characteristic finding in individuals with asbestosis.   While bronchoscopy, blood gas analysis, spirography, and electrocardiography may also be useful in evaluating lung function and identifying complications of asbestosis, such as lung cancer or pulmonary hypertension, they are not the most important method for diagnosing asbestosis.

Chest X-ray is the first-line diagnostic test for asbestosis and can provide important information about the extent and severity of the disease.   Therefore, in this case, the most important method for diagnosing asbestosis is chest X-ray.


65. A patient has got pain in the axillary area, rise of temperature developed 10 hours ago. On examination: shaky gait is evident, the tongue is coated with white deposit. The pulse is frequent. The painful lymphatic nodes are revealed in the axillary area. The skin over the lymph nodes is erythematous and glistering. What is the most probable diagnosis?

A. Bubonic plague

B. Acute purulent lymphadenitis

C. Lymphogranulomatosis

D. Anthrax

E. Tularemia


Answer: Bubonic plague

Explanation

The patient’s symptoms and physical findings suggest a diagnosis of bubonic plague, which is a bacterial infection caused by Yersinia pestis that is transmitted through the bite of infected fleas.   The sudden onset of pain in the axillary area, along with the rise of temperature and shaky gait, are all common symptoms of bubonic plague.

The presence of painful lymph nodes in the axillary area, along with erythema and blistering of the overlying skin, are characteristic of bubonic plague. The coated white tongue and frequent pulse may also be present in individuals with severe cases of the disease.  

Acute purulent lymphadenitis, lymphogranulomatosis, and anthrax can also cause painful lymph nodes and skin changes, but the sudden onset of symptoms and the presence of erythema and blistering are more suggestive of bubonic plague.

Tularemia can also cause lymphadenopathy, but it typically presents with fever and other systemic symptoms, such as headache and muscle aches.   Therefore, based on the patient’s symptoms and physical findings, the most probable diagnosis is bubonic plague. It is important to note that bubonic plague is a rare disease, but it can be life-threatening if not treated promptly with antibiotics.


66. Examination of a 9 month old girl revealed skin pallor, cyanosis during excitement. Percussion revealed transverse dilatation of cardiac borders. Auscultation revealed continuous systolic murmur on the left from the breastbone in the 3-4 intercostal space. This murmur is conducted above the whole cardiac region to the back. What congenital cardiac pathology can be suspected?

A. Defect of interventricular septum

B. Defect of interatrial septum

C. Coarctation of aorta

D. Fallot’s tetrad

E. Pulmonary artery stenosis


Answer: Defect of interventricular septum

Explanation

The patient’s symptoms and physical findings suggest a diagnosis of a defect of interventricular septum, which is a congenital heart defect in which there is a hole in the wall that separates the two lower chambers of the heart.  

The skin pallor and cyanosis during excitement are indicative of poor oxygenation, which can occur in individuals with a congenital heart defect. The transverse dilatation of cardiac borders and continuous systolic murmur on the left from the breastbone in the 3-4 intercostal space are both consistent with a defect of interventricular septum. The murmur is conducted above the whole cardiac region to the back, indicating that it is a large defect.  

Defects of interatrial septum, coarctation of aorta, Fallot’s tetrad, and pulmonary artery stenosis can also cause congenital heart defects and present with similar symptoms and physical findings. However, the continuous systolic murmur on the left from the breastbone in the 3-4 intercostal space points more towards a defect of interventricular septum.  


Therefore, based on the patient’s symptoms and physical findings, a defect of interventricular septum is the most likely diagnosis. The treatment for this condition may include surgical repair to close the hole in the heart.


67. A 32 year old patient complains about cardiac irregularities, dizziness, dyspnea at physical stress. He has never suffered from this before. Objectively: Ps- 74 bpm, rhythmic. AP- 130/80 mm Hg. Auscultation revealed systolic murmur above aorta, the first heart sound was normal. ECG showed hypertrophy of the left ventricle, signs of repolarization disturbance in the I, V5 and V6 leads. Echocardiogram revealed that interventricular septum was 2 cm. What is the most probable diagnosis?

A. Hypertrophic cardiomyopathy

B. Aortic stenosis

C. Essential hypertension

D. Myocardium infarction

E. Coarctation of aorta


Answer: Hypertrophic cardiomyopathy

 Explanation

The patient’s symptoms and physical findings suggest a diagnosis of hypertrophic cardiomyopathy, which is a genetic condition characterized by an abnormal thickening of the heart muscle, most commonly affecting the left ventricle.   The patient’s complaints of cardiac irregularities, dizziness, and dyspnea at physical stress, along with the auscultation findings of a systolic murmur above the aorta, are all consistent with hypertrophic cardiomyopathy.

The ECG findings of left ventricular hypertrophy and repolarization disturbances in the I, V5, and V6 leads are also characteristic of hypertrophic cardiomyopathy. The echocardiogram finding of a 2 cm interventricular septum confirms the diagnosis.  

Aortic stenosis and coarctation of the aorta can also cause a systolic murmur above the aorta and left ventricular hypertrophy. However, hypertrophic cardiomyopathy is the most common cause of left ventricular hypertrophy in young adults, and the presence of a normal first heart sound is more suggestive of hypertrophic cardiomyopathy than aortic stenosis or coarctation of the aorta. Essential hypertension can also cause left ventricular hypertrophy, but it does not typically present with a systolic murmur or repolarization disturbances on ECG.  

Myocardial infarction is unlikely in this case because the patient’s symptoms and physical findings are not consistent with acute coronary syndrome.   Therefore, based on the patient’s symptoms, physical findings, and diagnostic tests, the most probable diagnosis is hypertrophic cardiomyopathy. Treatment for hypertrophic cardiomyopathy may include medications to control symptoms and prevent complications, as well as surgical interventions such as septal myectomy or alcohol septal ablation in severe cases.


68. An 8 year old boy suffering from haemophilia was undergoing transfusion of packed red cells. Suddenly he got pain behind the breastbone and in the lumbar area, dyspnea, cold sweat. Objectively: pale skin, heart rate – 100/min, AP- 60/40 Hg; oliguria, brown urine. For treatment of this complication the following drug should be administered:

A. Prednisolone

B. Lasix

C. Adrenaline

D. Aminophylline

E. Analgine


Answer: Prednisolone

Explanation

The patient’s symptoms and physical findings suggest a diagnosis of acute hemolytic transfusion reaction, which is a rare but potentially life-threatening complication of blood transfusion. In this condition, the patient’s immune system reacts to transfused red blood cells, leading to the destruction of the cells and the release of hemoglobin into the bloodstream.  

The symptoms of pain behind the breastbone and in the lumbar area, dyspnea, cold sweat, pale skin, tachycardia, hypotension, oliguria, and brown urine are all consistent with acute hemolytic transfusion reaction. The brown urine is due to the presence of hemoglobin and its breakdown products, which can damage the kidneys and cause renal failure.  


The treatment for acute hemolytic transfusion reaction involves stopping the transfusion immediately, providing supportive care, and administering medications to manage the symptoms. Prednisolone, a corticosteroid, is often given to reduce inflammation and prevent further destruction of red blood cells. Lasix, a diuretic, may be used to manage fluid overload and oliguria, but it is not the most important drug in this situation.

Adrenaline and aminophylline are not typically used in the treatment of acute hemolytic transfusion reaction. Analgine, a painkiller, may be used to manage pain, but it is not the most important drug in this situation.   Therefore, based on the patient’s symptoms and physical findings, the most appropriate drug to administer for the treatment of acute hemolytic transfusion reaction is prednisolone.


69. A maternity hospital registered 616 live births, 1 stillbirth, 1 death on the 5th day of life over a 1 year period. What index allows the most precise estimation of this situation?

A. Perinatal mortality

B. Crude mortality rate

C. Natality

D. Neonatal mortality

E. Natural increase


Answer: Perinatal mortality

Explanation

Perinatal mortality is a measure of the number of stillbirths and deaths in the first week of life per 1,000 live births. It is an important index for assessing the quality of maternal and neonatal care in a hospital or community.  

In this case, the hospital registered 616 live births, 1 stillbirth, and 1 death on the 5th day of life over a 1 year period. The perinatal mortality rate can be calculated as follows:   Perinatal mortality rate = (Number of stillbirths + Number of neonatal deaths) / Total number of live births x 1,000   In this case, the perinatal mortality rate would be:   (1 stillbirth + 1 neonatal death) / 616 live births x 1,000 = 3.25 per 1,000 live births  

This index allows for the most precise estimation of the situation in the hospital because it takes into account both stillbirths and neonatal deaths, which are the two most common causes of perinatal mortality. The crude mortality rate, natality, neonatal mortality, and natural increase are not as appropriate for assessing the quality of maternal and neonatal care in this hospital.


70. After objective clinical examination a 12 year old child was diagnosed with mitral valve prolapse. What complementary instrumental method of examination should be applied for the diagnosis confi- rmation?

A. Echocardiography

B. Roentgenography of chest

C. Phonocardiography

D. ECG

E. Veloergometry


Answer: Echocardiography

Explanation

Mitral valve prolapse is a condition in which the valve between the left atrium and left ventricle of the heart does not close properly, allowing blood to leak back into the atrium during systole. The diagnosis of mitral valve prolapse is typically made based on the patient’s symptoms and physical findings, which may include a systolic click and/or a late systolic murmur heard on auscultation.  

However, to confirm the diagnosis of mitral valve prolapse and evaluate the severity of the condition, a complementary instrumental method of examination is needed. Echocardiography is the most appropriate method for this purpose.

Echocardiography uses ultrasound waves to produce images of the heart and its structures, including the mitral valve. It can show the degree of prolapse, the amount of regurgitation (leakage), and any other associated abnormalities.  

Roentgenography of chest, phonocardiography, ECG, and veloergometry are not typically used to diagnose mitral valve prolapse. Roentgenography of chest may show an enlarged left atrium or ventricle, which can be a sign of mitral regurgitation, but it is not specific to mitral valve prolapse. Phonocardiography and ECG may show a systolic click or late systolic murmur, but they cannot provide detailed information about the mitral valve itself.

Veloergometry is a test used to assess cardiovascular fitness and is not related to the diagnosis of mitral valve prolapse.   Therefore, based on the patient’s diagnosis of mitral valve prolapse, the most appropriate complementary instrumental method of examination for diagnosis confirmation is echocardiography.


71. A 28 year old parturient complains about headache, vision impairment, psychic inhibition. Objectively: AP200/110 mm Hg, evident edemata of legs and anterior abdominal wall. Fetus head is in the area of small pelvis. Fetal heartbeats is clear, rhythmic, 190/min. Internal investigation revealed complete cervical dilatation, fetus head was in the area of small pelvis. What tactics of labor management should be chosen?

A. Forceps operation

B. Cesarean

C. Embryotomy

D. Conservative labor management with episiotomy

E. Stimulation of labor activity


Answer: Forceps operation

Explanation

The patient’s symptoms and physical findings suggest a diagnosis of severe preeclampsia, which is a serious complication of pregnancy characterized by high blood pressure, proteinuria, and end-organ damage. The symptoms of headache, vision impairment, and psychic inhibition, along with the evident edema of legs and anterior abdominal wall, are all consistent with severe preeclampsia.  

The internal investigation revealed complete cervical dilatation and the fetus head in the area of small pelvis, indicating that the patient is in labor. However, in the setting of severe preeclampsia, it is usually safer to deliver the baby via cesarean section rather than vaginal delivery with forceps or other interventions.  

Forceps operation or other interventions can increase the risk of bleeding, trauma, and other complications, which can be particularly dangerous in the setting of severe preeclampsia. Cesarean section allows for a controlled delivery of the baby and can reduce the risk of complications for both the mother and baby.  

Therefore, based on the patient’s symptoms and physical findings, the most appropriate tactic of labor management is cesarean section. It is important to deliver the baby as soon as possible to prevent further complications associated with severe preeclampsia.


72. A patient complained about problems with pain and tactile sensitivity, pain in the nail bones at the end of the working day. He works at a plant with mechanical devices. What pathology can be suspected?

A. Vibration disease

B. Caisson disease

C. Noise disease

D. Overwork symptoms

E. Hypovitaminosis of B1


Answer: Vibration disease

Explanation

The patient’s symptoms of pain and tactile sensitivity, particularly in the nail bones at the end of the working day, suggest a diagnosis of vibration disease. Vibration disease, also known as hand-arm vibration syndrome, is a condition caused by prolonged exposure to mechanical vibrations, such as those from working with power tools, machinery, or other vibrating devices.  

The symptoms of vibration disease typically include numbness, tingling, and pain in the fingers, hands, and arms, as well as a reduced ability to detect touch and temperature. The symptoms may be worse at the end of the working day or on weekends, and they may improve with rest.  

Caisson disease is a condition caused by exposure to high-pressure environments, such as those experienced by deep-sea divers or workers in pressurized environments. The symptoms of caisson disease typically include joint pain, muscle weakness, and neurological symptoms such as numbness, tingling, and paralysis.  

Noise disease, also known as noise-induced hearing loss, is a condition caused by exposure to loud noises over a prolonged period of time. The symptoms of noise disease typically include hearing loss and tinnitus (ringing in the ears).  

Overwork symptoms and hypovitaminosis of B1 are not specific diagnoses and do not explain the patient’s symptoms.   Therefore, based on the patient’s symptoms and occupational history, the most probable diagnosis is vibration disease. The patient should be advised to reduce or eliminate exposure to mechanical vibrations, and may benefit from treatment with medications to alleviate pain and improve sensory function.


73. A 25 year old patient complains about weakness, dizziness, haemorrhagic skin rash. She has been suffering from this for a month. Blood count: erythrocytes: 1, 0 · 1012/l, Hb- 37 g/l, colour index – 0,9, leukocytes – 1, 2 · 109/l, thrombocytes – 42 · 109/l. What diagnostic method will be the most effective?

A. Sternal puncture

B. Spleen biopsy

C. Liver biopsy

D. Coagulogram

E. Abdominal ultrasound


Answer: Sternal puncture

Explanation

The patient’s symptoms and blood count suggest a diagnosis of aplastic anemia, which is a rare but serious condition characterized by a deficiency of all types of blood cells (red blood cells, white blood cells, and platelets) due to damage to the bone marrow. The symptoms of weakness, dizziness, and hemorrhagic skin rash, along with the low blood counts of erythrocytes, leukocytes, and thrombocytes, are all consistent with aplastic anemia.  

To confirm the diagnosis of aplastic anemia and assess the severity of the condition, a sternal puncture (also known as a bone marrow biopsy) is the most effective diagnostic method. A sternal puncture involves the insertion of a needle into the bone marrow in the breastbone (sternum) to obtain a sample of bone marrow tissue. The sample is then examined under a microscope to evaluate the number, size, and shape of blood cells and determine the cause of the low blood counts.  

Spleen biopsy, liver biopsy, coagulogram, and abdominal ultrasound are not typically used to diagnose aplastic anemia. Spleen and liver biopsies may be used to diagnose other conditions, such as liver disease or cancer, but they are not specific to aplastic anemia.

Coagulogram is a test used to evaluate blood clotting function, which may be affected in some types of anemia but is not necessary for diagnosing aplastic anemia. Abdominal ultrasound may be used to evaluate the spleen, liver, and other abdominal organs, but it is not the most effective diagnostic method for aplastic anemia.   Therefore, based on the patient’s symptoms and blood count, the most effective diagnostic method for confirming the diagnosis of aplastic anemia and assessing the severity of the condition is sternal puncture.


74. A 68 year old female patient complains about temperature rise up to 38, 3oC, haematuria. ESR- 55 mm/h. Antibacterial therapy turned out to be neffective. What diagnosis might be suspected?

A. Renal cancer

B. Polycystic renal disease

C. Renal amyloidosis

D. Urolithiasis

E. Chronic glomerulonephritis

Answer: Renal cancer

Explanation

The patient’s symptoms of fever, hematuria, and elevated ESR, along with the lack of response to antibacterial therapy, suggest a possible diagnosis of renal cancer. Renal cancer is a type of cancer that originates in the cells of the kidney and can cause a variety of symptoms, including hematuria, fever, and weight loss.  

Polycystic renal disease is a genetic disorder characterized by the growth of multiple cysts in the kidneys, which can cause kidney failure, high blood pressure, and other complications. However, it is unlikely to present with the acute onset of symptoms described in this case.  

Renal amyloidosis is a rare condition in which proteins called amyloids build up in the kidneys, causing damage and reduced kidney function. It may present with hematuria, proteinuria, and other symptoms, but it is not typically associated with fever or an elevated ESR.  

Urolithiasis, also known as kidney stones, is a condition in which solid masses of crystals form in the kidneys or urinary tract, causing pain, hematuria, and other symptoms. However, it is unlikely to cause an elevated ESR or fever.  

Chronic glomerulonephritis is a type of kidney disease in which the glomeruli, the tiny structures in the kidneys that filter waste from the blood, become inflamed and damaged over time. It may present with hematuria, proteinuria, and other symptoms, but it is not typically associated with fever or an elevated ESR.  

Therefore, based on the patient’s symptoms and laboratory findings, the most probable diagnosis is renal cancer. Additional diagnostic tests, such as imaging studies or a renal biopsy, may be needed to confirm the diagnosis and determine the extent of the disease.


75. A 48 year old female patient complains about contact haemorrhage. Speculum examination revealed hypertrophy of uterus cervix. It resembles of cauliflower, it is dense and can be easily injured. Bimanual examination revealed that fornices were shortened, uterine body was nonmobile. What is the most probable diagnosis?

A. Cervical carcinoma

B. Metrofibroma

C. Endometriosis

D. Cervical pregnancy

E. Cervical papillomatosis


Answer: Cervical carcinoma

Explanation

The patient’s symptoms of contact bleeding and the appearance of the cervix on speculum examination suggest a diagnosis of cervical carcinoma. Cervical carcinoma is a type of cancer that originates in the cells of the cervix and can cause abnormal vaginal bleeding, pain, and other symptoms. The hypertrophy of the cervix that resembles a cauliflower, is dense, and easily injured is a characteristic finding of cervical cancer.  

Metrofibroma, also known as uterine fibroids, is a common benign tumor of the uterus that can cause abnormal vaginal bleeding, pain, and other symptoms. However, it typically does not cause the characteristic appearance of the cervix seen in this case.  

Endometriosis is a condition in which tissue similar to the lining of the uterus grows outside of the uterus, causing pain and other symptoms. It is not typically associated with the appearance of the cervix seen in this case.   Cervical pregnancy is a rare and serious condition in which the fetus develops in the cervix instead of the uterus, causing vaginal bleeding and other complications. It is unlikely to present with the characteristic appearance of the cervix seen in this case.  

Cervical papillomatosis is a benign condition in which there is an overgrowth of cells on the surface of the cervix, causing a wart-like appearance. It is not typically associated with contact bleeding or the dense, cauliflower-like hypertrophy of the cervix seen in this case.  

Therefore, based on the patient’s symptoms and physical examination findings, the most probable diagnosis is cervical carcinoma. Additional diagnostic tests, such as a cervical biopsy or imaging studies, may be needed to confirm the diagnosis and determine the extent of the disease.


76. A 34 year old female patient has been suffering from anxious depression accompanied by panic attacks for 2 years. She has been a patient of a psychotherapist. Treatment resulted in incomplete remission. The patient had to break off psychotherapy because of moving to a new place of residence. Soon after this her condition grew significantly worse, she was almost permanently anxious, panic attacks turned up 5-6 times a day and were accompanied by palpitation, dyspnea, cold sweat, thanatophobia. What drug group is the most appropriate for medicamental therapy?

A. Antidepressants

B. Sedative neuroleptics

C. Antipsychotic neuroleptics

D. Lithium drugs

E. Cardiotonics, respiratory analeptics


Answer: Antidepressants

Explanation

The patient’s symptoms of anxious depression and panic attacks suggest a diagnosis of panic disorder, which is a type of anxiety disorder characterized by recurrent, unexpected panic attacks. The patient’s worsening symptoms after discontinuing psychotherapy suggest that she may benefit from medication management.  

Antidepressants, specifically selective serotonin reuptake inhibitors (SSRIs) and serotonin-norepinephrine reuptake inhibitors (SNRIs), are the most appropriate drug group for the medicamental therapy of panic disorder.

These medications work by increasing the levels of serotonin and norepinephrine in the brain, which can help to reduce anxiety symptoms and prevent panic attacks.   Sedative neuroleptics, antipsychotic neuroleptics, lithium drugs, and cardiotonics and respiratory analeptics are not appropriate for the treatment of panic disorder.


Sedative neuroleptics and antipsychotic neuroleptics are typically used to treat psychotic disorders and are not effective in treating anxiety symptoms.

Lithium drugs are primarily used to treat bipolar disorder and are not effective in treating panic disorder. Cardiotonics and respiratory analeptics are used to treat cardiovascular and respiratory conditions, respectively, and are not effective in treating anxiety symptoms.  

Therefore, based on the patient’s symptoms and history, the most appropriate drug group for medicamental therapy is antidepressants, specifically SSRIs or SNRIs. The patient should be evaluated by a psychiatrist or other mental health professional to determine the most appropriate medication and dosage.


77. 1,5 hour after start of gullet bougienage a 48 year old patient suffering from corrosive stricture felt acute abdominal pain. Previously he had been suffering from duodenal ulcer. Examination revealed that abdomen was very tense and painful; Ps- 110 bpm, painful sialophagia, skin pallor. What is the most probable diagnosis?

A. Perforation of abdominal part of esophagus

B. Acute myocardium infarction

C. Strangulation of diaphragmal hernia

D. Perforation of duodenal ulcer

E. Thrombosis of mesenteric vessels


Answer: Perforation of abdominal part of esophagus

Explanation

The patient’s symptoms of acute abdominal pain, tense and painful abdomen, tachycardia, painful sialophagia, and skin pallor suggest a diagnosis of perforation of the abdominal part of the esophagus. Corrosive strictures can develop in the esophagus as a result of long-standing acid reflux or ingestion of caustic substances, which can weaken the wall of the esophagus and make it prone to perforation during invasive procedures, such as gullet bougienage.  

Acute myocardial infarction, or heart attack, is a medical emergency that typically presents with chest pain, shortness of breath, and other symptoms related to the heart. It is not likely to present with the symptoms described in this case.   Strangulation of diaphragmatic hernia occurs when a portion of the stomach or other abdominal organs become trapped in the diaphragmatic opening, causing symptoms such as abdominal pain, nausea, and vomiting.

However, it is unlikely to occur during gullet bougienage and is not typically associated with painful sialophagia.   Perforation of duodenal ulcer can cause sudden onset of severe abdominal pain, but it is not typically associated with painful sialophagia or skin pallor.   Thrombosis of mesenteric vessels, or mesenteric ischemia, can cause severe abdominal pain, nausea, and vomiting due to reduced blood flow to the intestines. However, it is unlikely to occur during gullet bougienage and typically presents with other symptoms, such as diarrhea and bloody stools.  

Therefore, based on the patient’s symptoms and history, the most probable diagnosis is perforation of the abdominal part of the esophagus. This is a medical emergency that requires immediate attention, and the patient should be referred to a specialist for further evaluation and treatment.


78. A healthy 75 year old woman who leads a moderately active way of life went through a preventive examination that revealed serum concentration of common cholesterol at the rate of 5,1 millimole/l and HDL (high-density lipoproteins) cholesterol at the rate of 70 mg/dl. ECG reveals no pathology. What dietary recommendation is the most adequate?

A. Any dietary changes are necessary

B. Decrease of cholesterol consumption

C. Decrease of saturated fats consumption

D. Decrease of carbohydrates consumption

E. Increase of cellulose consumption


Answer: Any dietary changes are necessary

Explanation

The patient’s total cholesterol level is slightly above the recommended level of below 5 millimoles per liter (mmol/L), but her HDL cholesterol level is within the recommended range of above 60 milligrams per deciliter (mg/dL) for women.

Additionally, her ECG is normal and there is no indication of other risk factors for cardiovascular disease. Therefore, no specific dietary changes are necessary at this time.   While reducing cholesterol and saturated fat intake and increasing fiber intake can have cardiovascular benefits, the patient’s current cholesterol and HDL levels do not warrant such strict dietary changes.

In fact, a moderate intake of healthy fats, such as those found in nuts, seeds, and fatty fish, may even be beneficial for cardiovascular health.   It is important for the patient to maintain a healthy and balanced diet, as well as engage in regular physical activity, to maintain her overall health and reduce the risk of developing cardiovascular disease in the future.

However, based on the results of her preventive examination, no specific dietary recommendations are necessary at this time.  

Therefore, the most appropriate dietary recommendation for this patient is that any dietary changes are not necessary, but she should maintain a healthy and balanced diet and engage in regular physical activity to maintain her overall health.


79. A parturient complains about pain in the mammary gland. Palpation revealed a 3х4 cm large infiltration, soft in the centre. Body temperature is 38, 5oC. What is the most probable diagnosis?

A. Acute purulent mastitis

B. Pneumonia

C. Pleuritis

D. Retention of milk

E. Birth trauma


Answer: Acute purulent mastitis

Explanation

The patient’s symptoms of pain in the mammary gland, a palpable infiltration, and a high body temperature suggest a diagnosis of acute purulent mastitis. This is a bacterial infection of the breast tissue that can occur in lactating women, typically within the first few weeks after delivery.

The infection can cause pain, redness, swelling, and warmth in the affected breast, as well as fever and malaise.   Pneumonia is a respiratory infection that can cause symptoms such as cough, fever, and difficulty breathing, but it typically does not cause breast pain or palpable infiltration.  

Pleuritis, or inflammation of the lining of the lungs, can cause chest pain, fever, and difficulty breathing, but it does not typically cause breast pain or palpable infiltration.   Retention of milk, or engorgement, can cause breast pain and swelling, but it typically does not cause a palpable infiltration or high body temperature.  

Birth trauma, or injury to the breast tissue during delivery, can cause breast pain and tenderness, but it typically does not cause a palpable infiltration or high body temperature.   Therefore, based on the patient’s symptoms and physical examination findings, the most probable diagnosis is acute purulent mastitis.

The patient should be evaluated by a healthcare provider and treated with antibiotics and other supportive measures, such as warm compresses and pain relief medication, as needed.


80. A 52 year old patient was admitted to a hospital because of high hemorrhagic diathesis of mucous membranes, massive skin haemorrhages in form of ecchymoses and spots, nasal and stomachal haemorrhages. After clinical examinations her illness was diagnosed as thrombocytopenic purpura. What is the most probable cause of this disease?

A. Generation of antithrombocytic antibodies

B. Disturbed hemostasis

C. Deficit of the VIII factor of blood coagulation

D. Inherited insufficiency of plasm factors of blood coagulation

E. Iron deficit in blood serum, bone marrow and depot


Answer:  Generation of antithrombocytic antibodies

Explanation

Thrombocytopenic purpura is a condition characterized by a low platelet count and bleeding tendency, resulting in the formation of skin hemorrhages, mucous membrane bleeding, and other bleeding symptoms. The most probable cause of this disease is the generation of antithrombocytic antibodies, which are antibodies that target and destroy platelets in the blood.   Antithrombocytic antibodies can be produced in response to various factors, such as viral infections, medications, autoimmune disorders, or unknown causes.

These antibodies can bind to the platelets and cause their destruction, leading to thrombocytopenia and bleeding symptoms.   Disturbed hemostasis, deficit of the VIII factor of blood coagulation, inherited insufficiency of plasm factors of blood coagulation, and iron deficit in blood serum, bone marrow, and depot are not the most probable causes of thrombocytopenic purpura.   Disturbed hemostasis can cause bleeding disorders, but it typically does not result in thrombocytopenia.  

Deficit of the VIII factor of blood coagulation is associated with hemophilia A, a bleeding disorder characterized by deficiency or dysfunction of this clotting factor, but it does not typically cause thrombocytopenia.   Inherited insufficiency of plasm factors of blood coagulation can cause bleeding disorders, but it does not typically cause thrombocytopenia.   Iron deficit in blood serum, bone marrow, and depot can cause anemia, but it is not typically associated with thrombocytopenic purpura.  

Therefore, based on the patient’s symptoms and clinical examination findings, the most probable cause of thrombocytopenic purpura is the generation of antithrombocytic antibodies. The patient should be evaluated and treated by a hematologist or other appropriate specialist.


81. A 37 year old patient applied to a local therapeutist. As a result of exacerbation of chronic obstructive bronchitis the patient had been temporarily disabled for 117 days within 1 year. What tactics will be legally correct?

A. The patient should be referred to the medicosocial expertise

B. The therapeutist should extend a medical certificate

C. The patient should be referred to the medical consultation comission for extension of medical certificate

D. The therapeutist should issue a new medical certificate

E. The patient shoul be referred to the sanatorium-and-spa treatment


Answer: The patient should be referred to the medicosocial expertise

Explanation

The patient’s temporary disability for 117 days within 1 year due to exacerbation of chronic obstructive bronchitis suggests that the patient may have a chronic medical condition that affects their ability to work. In this case, it is legally correct to refer the patient to the medicosocial expertise for evaluation of their disability status and determination of appropriate measures to support their health and work capacity.  

Medicosocial expertise is a process by which a patient’s medical condition and functional abilities are evaluated by a team of medical and social experts. The aim of this process is to determine the degree of disability, if any, and to provide recommendations for appropriate measures to support the patient’s health and work capacity, such as rehabilitation, vocational training, or workplace accommodations.  

The therapeutist may extend a medical certificate or issue a new one, but this is not sufficient in cases where a patient’s chronic medical condition affects their ability to work and requires further evaluation by the medicosocial expertise.   Referring the patient to the medical consultation commission or to the sanatorium-and-spa treatment may be appropriate in certain cases, but it is not the most appropriate course of action in this case, where the patient’s chronic medical condition requires evaluation of their disability status and appropriate measures to support their health and work capacity.   Therefore, the most legally correct tactic in this case is to refer the patient to the medicosocial expertise.


82. A 54 year old male patient complains about permanent dull pain in the mesogastral region, weight loss, dark blood admixtures in the feces, constipations. He put off 10 kg within a year. In blood: erythrocytes: 3, 5·1012/l, Hb- 87 g/l, leukocytes – 12, 6 · 109/l, stab neutrophil shift, ESR- 43 mm/h. What is the most probable diagnosis?

A. Cancer of transverse colon

B. Gastric ulcer

C. Chronic colitis

D. Chronic pancreatitis

E. Stomach cancer


Answer:  Cancer of transverse colon

Explanation

The patient’s symptoms of permanent dull pain in the mesogastral region, weight loss, dark blood admixtures in the feces, constipation, and significant loss of weight within a year, along with laboratory findings of anemia, leukocytosis with a shift to the left, and elevated ESR, suggest a diagnosis of cancer of the transverse colon.  

Gastric ulcer can cause abdominal pain, but it typically does not cause weight loss or dark blood admixtures in the feces.   Chronic colitis can cause abdominal pain, diarrhea, and blood in the stool, but it typically does not cause constipation or significant weight loss.  

Chronic pancreatitis can cause abdominal pain, weight loss, and digestive problems, but it typically does not cause dark blood admixtures in the feces.   Stomach cancer can cause abdominal pain, weight loss, and digestive problems, but it typically does not cause dark blood admixtures in the feces or constipation.  

Therefore, based on the patient’s symptoms and laboratory findings, the most probable diagnosis is cancer of the transverse colon. The patient should be evaluated by a gastroenterologist and undergo further diagnostic tests, such as colonoscopy or CT scan, to confirm the diagnosis and plan appropriate treatment, which may include surgery, chemotherapy, and/or radiation therapy. Early detection and treatment of colon cancer can improve the patient’s prognosis and survival.


83. A 10 month old boy has been ill for 5 days after consumption of unboiled milk. Body temperature is 38 − 39oC, there is vomiting, liquid stool. The child is pale and inert. His tongue is covered with white deposition. Heart sounds are muffled. Abdomen is swollen, there is borborygmus in the region of ubbilicus, liver is enlarged by 3 cm. Stool is liquid, dark-green, with admixtures of mucus, 5 times a day. What is the most probable diagnosis?

A. Salmonellosis

B. Staphylococcal enteric infection

C. Escherichiosis

D. Acute shigellosis

E. Rotaviral infection


Answer:  Salmonellosis

Explanation

The patient’s symptoms of fever, vomiting, diarrhea, pale appearance, white deposition on the tongue, muffled heart sounds, swollen abdomen with borborygmus and liver enlargement, and dark-green liquid stool with mucus suggest a diagnosis of salmonellosis.  

Salmonellosis is a bacterial infection caused by the Salmonella bacteria commonly found in contaminated food, especially uncooked or undercooked meat and dairy products. Symptoms of salmonellosis include fever, abdominal pain, vomiting, and diarrhea, often with blood or mucus in the stool.   Staphylococcal enteric infection typically causes symptoms such as nausea, vomiting, and diarrhea, but it typically does not cause fever or liver enlargement.  

Escherichiosis, or infection with Escherichia coli bacteria, can cause diarrhea, abdominal pain, and fever, but it typically does not cause liver enlargement or muffled heart sounds.   Acute shigellosis, or infection with Shigella bacteria, can cause symptoms such as fever, abdominal pain, and bloody diarrhea, but it typically does not cause liver enlargement or muffled heart sounds.  

Rotaviral infection can cause symptoms such as fever, vomiting, and diarrhea, but it typically does not cause liver enlargement or muffled heart sounds.   Therefore, based on the patient’s symptoms and physical examination findings, the most probable diagnosis is salmonellosis. The patient should be evaluated by a healthcare provider and treated with appropriate antibiotics and supportive measures, such as fluid and electrolyte replacement, as needed.


84. Examination of a 22 year old man suffering from polyarthralgia and high fever revealed right-sided exudative pleuritis. X-ray picture showed a homogenous shadow below the IV rib on the right. In the II segment there were single dense focal shadows. Mantoux test with 2 TU resulted in formation of a papula 16 mm large. Pleural liquid has increased protein concentration, Rivalta’s reaction is positive, there was also increased number of leukocytes with prevailing lymphocytes. What is the most probable etiology of pleuritis?

A. Tuberculous

B. Cancerous

C. Staphylococcal

D. Viral

E. Autoimmune


Answer: Tuberculous

Explanation

The patient’s symptoms of polyarthralgia, high fever, and right-sided exudative pleuritis, along with radiographic findings of a homogenous shadow below the IV rib on the right and single dense focal shadows in the II segment, suggest a diagnosis of tuberculous pleuritis.  

The positive Mantoux test with a large papula size and pleural fluid analysis showing increased protein concentration, positive Rivalta’s reaction, and increased number of leukocytes with prevailing lymphocytes further support the diagnosis of tuberculous pleuritis.  

Cancerous pleuritis can cause fluid accumulation in the pleural space, but it typically does not cause polyarthralgia or a positive Mantoux test.   Staphylococcal pleuritis can cause fluid accumulation in the pleural space, but it typically does not cause a positive Mantoux test or increased number of lymphocytes in pleural fluid analysis.  

Viral pleuritis can cause fluid accumulation in the pleural space, but it typically does not cause a positive Mantoux test or increased protein concentration in pleural fluid analysis.   Autoimmune pleuritis, such as lupus pleuritis, can cause fluid accumulation in the pleural space, but it typically does not cause a positive Mantoux test or increased protein concentration in pleural fluid analysis.  

Therefore, based on the patient’s symptoms, radiographic findings, Mantoux test result, and pleural fluid analysis, the most probable etiology of pleuritis is tuberculous. The patient should be evaluated by a pulmonologist or infectious disease specialist and undergo further diagnostic tests, such as sputum culture or PCR analysis, to confirm the diagnosis and plan appropriate treatment, which typically includes a combination of antibiotics for several months.


85. At year-end hospital administration has obtained the following data: annual number of treated patients and average annual number of beds used for patients’ treatment. What index of hospital work can be calculated on the base of this data?

A. Bed turnover

B. Bed resources of the hospital

C. Average annual bed occupancy

D. Average duration of patients’ presence in the hospital

E. Average bed idle time


Answer: Bed turnover

Explanation

Bed turnover is an index of hospital work that measures the number of times a hospital bed is occupied by different patients over a given period of time, usually a year. It is calculated by dividing the total number of patients treated in the hospital during the year by the average number of beds used for patient treatment during the same period.  

Bed turnover = (Total number of patients treated during the year) / (Average number of beds used for patient treatment during the year)   Bed resources of the hospital refers to the total number of beds available in the hospital, which is not the same as the average number of beds used for patient treatment.   Average annual bed occupancy measures the percentage of time that hospital beds are occupied by patients over a year, and it is calculated by dividing the total number of occupied bed-days by the total number of available bed-days during the same period.  

Average duration of patients’ presence in the hospital measures the average length of stay of patients in the hospital, and it is calculated by dividing the total number of patient-days by the total number of patients.   Average bed idle time measures the percentage of time that hospital beds are unoccupied or not used for patient treatment over a year, and it is calculated by dividing the total number of unoccupied bed-days by the total number of available bed-days during the same period.  

Therefore, based on the given data, the index of hospital work that can be calculated is bed turnover, which measures the number of times a hospital bed is occupied by different patients over a given period of time.


86. A female patient has been suffering from pain in the right subcostal area, bitter taste in the mouth, periodical bile vomiting for a month. The patient put off 12 kg. Body temperature in the evening is 37, 6oC. Sonography revealed that bile bladder was 5,5х2,7 cm large, its wall – 0,4 cm, choledochus – 0,8 cm in diameter. Anterior liver segment contains a roundish hypoechoic formation up to 5 cm in diameter and another two up to 1,5 cm each, walls of these formations are up to 0,3 cm thick. What is the most probable diagnosis?

A. Alveolar echinococcus of liver

B. Liver cancer

C. Liver abscess

D. Cystous liver cancer

E. Paravesical liver abscesses


Answer:  Alveolar echinococcus of liver

Explanation

The patient’s symptoms of pain in the right subcostal area, bitter taste in the mouth, periodical bile vomiting, and significant weight loss, along with laboratory findings of elevated body temperature in the evening, suggest a diagnosis of liver disease.  

Sonography findings of a 5.5 x 2.7 cm bile bladder, 0.4 cm wall, and 0.8 cm diameter choledochus, along with a roundish hypoechoic formation up to 5 cm in diameter and two others up to 1.5 cm each with walls up to 0.3 cm thick in the anterior liver segment, suggest a diagnosis of alveolar echinococcus of liver.  

Liver cancer can cause symptoms such as abdominal pain, weight loss, and elevated liver enzymes, but it typically does not cause bitter taste in the mouth or bile vomiting, and sonography findings may show a solid mass or nodules.   Liver abscess can cause symptoms such as fever, abdominal pain, and jaundice, but it typically does not cause bitter taste in the mouth or periodical bile vomiting, and sonography findings may show a fluid-filled cavity.  

Cystic liver cancer can cause symptoms such as abdominal pain and weight loss, but it typically does not cause bitter taste in the mouth or bile vomiting, and sonography findings may show a solid mass with cystic components.  

Paravesical liver abscesses can cause symptoms such as fever, abdominal pain, and jaundice, but they typically do not cause bitter taste in the mouth or periodical bile vomiting, and sonography findings may show a fluid-filled cavity adjacent to the liver.   Therefore, based on the patient’s symptoms and sonography findings, the most probable diagnosis is alveolar echinococcus of the liver, which is a parasitic infection caused by the tapeworm Echinococcus multilocularis.

The patient should be referred to a specialist in tropical medicine or infectious diseases and undergo further diagnostic tests, such as serology or biopsy, to confirm the diagnosis and plan appropriate treatment, which may include surgery, chemotherapy, or a combination of both. Early detection and treatment of alveolar echinococcus of the liver can improve the patient’s prognosis and prevent complications.


87. A 3 year old child with weight defi- ciency suffers from permanent moist cough. In history there are some pneumonias with obstruction. On examination: distended chest, dullness on percussion over the lower parts of lungs. On auscultation: a great number of different rales. Level of sweat chloride is 80 millimol/l. What is the most probable diagnosis?

A. Mucoviscidosis (cystic fibrosis)

B. Bronchial asthma

C. Recurrent bronchitis

D. Bronchiectasis

E. Pulmonary hypoplasia


Answer:  Mucoviscidosis (cystic fibrosis)

Explanation

The child’s symptoms of weight deficiency, permanent moist cough, history of pneumonias with obstruction, distended chest, dullness on percussion over the lower parts of lungs, a great number of different rales on auscultation, and elevated sweat chloride level suggest a diagnosis of mucoviscidosis, also known as cystic fibrosis.   Mucoviscidosis is a genetic disorder that affects multiple organs, including the lungs, pancreas, and sweat glands.

It is characterized by thick, sticky mucus that clogs the airways, leading to chronic respiratory infections, bronchiectasis, and eventually, respiratory failure.   Bronchial asthma can cause symptoms such as coughing, wheezing, and shortness of breath, but it typically does not cause weight deficiency, dullness on percussion, or elevated sweat chloride level.   Recurrent bronchitis can cause symptoms such as coughing and chest congestion, but it typically does not cause weight deficiency, dullness on percussion, or elevated sweat chloride level.  

Bronchiectasis can cause symptoms such as chronic cough, sputum production, and recurrent respiratory infections, but it typically does not cause weight deficiency or elevated sweat chloride level.   Pulmonary hypoplasia refers to underdevelopment of the lungs, which can occur in utero or after birth, and it typically presents with respiratory distress, cyanosis, and failure to thrive, but it does not cause chronic cough or elevated sweat chloride level.  

Therefore, based on the child’s symptoms and laboratory findings, the most probable diagnosis is mucoviscidosis, and the child should be referred to a specialist in pediatric pulmonology for further diagnostic tests, such as genetic testing and pulmonary function tests, and appropriate treatment, which may include airway clearance techniques, antibiotics, bronchodilators, and pancreatic enzyme replacement therapy. Early diagnosis and treatment of mucoviscidosis can improve the child’s prognosis and quality of life.


88. A 14 year old girl complains of profuse bloody discharges from genital tracts during 10 days after suppresion of menses for 1,5 month. Similiar bleedings recur since 12 years on the background of disordered menstrual cycle. On rectal examination: no pathology of the internal genitalia. In blood: Нb – 70 g/l, RBC2, 3 · 1012/l, Ht – 20. What is the most probable diagnosis?

A. Juvenile bleeding, posthemorrhagic anemia

B. Werlholf’s disease

C. Polycyst ovarian syndrome

D. Hormonoproductive ovary tumor

E. Incomplete spontaneous abortion


Answer: Juvenile bleeding, posthemorrhagic anemia

Explanation

The girl’s symptoms of profuse bloody discharges from the genital tracts, recurrence of similar bleedings since 12 years, and disordered menstrual cycle suggest a diagnosis of juvenile bleeding, also known as dysfunctional uterine bleeding. The suppression of menses for 1.5 months may have caused the buildup of the endometrium, leading to the profuse bleeding.   Posthemorrhagic anemia is a type of anemia that occurs after acute blood loss, such as from dysfunctional uterine bleeding.

The girl’s laboratory findings of Hb 70 g/l, RBC 2.3 x 10^12/l, and Ht 20% indicate a significant drop in hemoglobin and hematocrit levels, consistent with posthemorrhagic anemia.   Werlhof’s disease, also known as idiopathic thrombocytopenic purpura, is a bleeding disorder that is characterized by low platelet counts and easy bruising, but it typically does not cause profuse genital bleeding or disordered menstrual cycle.  

Polycystic ovarian syndrome is a hormonal disorder that is characterized by ovarian cysts, irregular menstrual cycles, and high levels of androgens, but it typically does not cause profuse genital bleeding.   Hormonoproductive ovary tumor is a rare type of ovarian tumor that secretes hormones, such as estrogen or androgen, and can cause menstrual irregularities, but it typically does not cause profuse genital bleeding.  

Incomplete spontaneous abortion is a type of miscarriage that occurs when the fetus and placenta are partially expelled from the uterus, but the remaining tissue continues to cause bleeding and cramping. However, this diagnosis is less likely given the girl’s history of recurrent similar bleedings since 12 years.  

Therefore, based on the girl’s symptoms and laboratory findings, the most probable diagnosis is juvenile bleeding, posthemorrhagic anemia. The girl should be referred to a gynecologist for further diagnostic tests, such as pelvic ultrasound and hormonal evaluation, and appropriate treatment, which may include hormonal contraceptives, nonsteroidal anti-inflammatory drugs (NSAIDs), or in severe cases, dilation and curettage (D&C) or hysterectomy. Early diagnosis and treatment of juvenile bleeding can prevent complications, such as anemia and infertility.


89. A 46 year old patient is to be prepared to the operation on account of stomach cancer. Preoperative preparation involves infusion therapy. It was injected up to 3 l of solutions into his right lunar vein. On the next day he got tensive pain in the region of his right shoulder. Examination of interior brachial surface revealed an oblong area of hyperemia, skin edema and painful cord. What complication is it?

A. Acute thrombophlebitis

B. Vein puncture and edema of paravenous cellular tissue

C. Necrosis of paravenous cellular tissue

D. Acute lymphangitis

E. Phlegmon of paravenous cellular tissue


Answer:  Acute thrombophlebitis

Explanation

The patient’s symptoms of tensive pain in the region of his right shoulder, an oblong area of hyperemia, skin edema, and painful cord on examination of the interior brachial surface suggest a diagnosis of acute thrombophlebitis, which is inflammation and clotting of a vein.  

Infusion therapy involves the administration of fluids and medications through a vein, which can cause irritation, injury, or infection of the vein, leading to thrombophlebitis. The right lunar vein is also known as the right cephalic vein, which runs along the anterior surface of the upper arm and drains into the axillary vein.  

Vein puncture and edema of paravenous cellular tissue can cause symptoms such as pain and swelling, but it typically does not cause skin hyperemia or a painful cord.   Necrosis of paravenous cellular tissue is a rare but serious complication that can occur when the infusion solution leaks into the surrounding tissue, causing tissue damage and death, but it typically presents with severe pain, skin discoloration, and systemic symptoms, such as fever and sepsis.  

Acute lymphangitis is inflammation and infection of the lymphatic vessels, which typically presents with red streaks on the skin, swollen lymph nodes, and systemic symptoms, such as fever and chills, but it does not cause a painful cord or skin edema.  

Phlegmon of paravenous cellular tissue is a spreading bacterial infection that can occur when the infusion solution contaminates the surrounding tissue, leading to tissue necrosis, abscess formation, and systemic symptoms, such as fever and sepsis, but it typically presents with diffuse swelling and redness, rather than an oblong area of hyperemia and painful cord.  

Therefore, based on the patient’s symptoms and history of infusion therapy, the most probable diagnosis is acute thrombophlebitis, and the patient should be referred to a specialist in vascular medicine or surgery for further diagnostic tests, such as Doppler ultrasound or venography, and appropriate treatment, which may include anticoagulants, pain relief, and warm compresses. Early diagnosis and treatment of acute thrombophlebitis can prevent complications, such as pulmonary embolism and chronic venous insufficiency.


90. A 58 year old female patient complains about periodical headache, dizziness and ear noise. She has been suffering from diabetes mellitus for 15 years. Objectively: heart sounds are rhythmic, heart rate is 76/min, there is diastolic shock above aorta, AP is 180/110 mm Hg. In urine: OD- 1,014. Daily loss of protein with urine is 1,5 g. What drug should be chosen for treatment of arterial hypertension?

A. Ihibitor of angiotensin converting enzyme

B. β-blocker

C. Calcium channel antagonist

D. Thiazide diuretic

E. α-blocker


Answer: Ihibitor of angiotensin converting enzyme 

Explanation

The patient’s symptoms of periodical headache, dizziness, and ear noise, as well as her history of diabetes mellitus, suggest a diagnosis of hypertension, which is a common complication of diabetes mellitus.  

The patient’s objective findings of diastolic shock above the aorta, high blood pressure (AP) of 180/110 mm Hg, and proteinuria of 1.5 g/day suggest that she has hypertensive urgency or emergency, which requires immediate treatment to prevent complications, such as stroke, heart attack, or kidney failure.  

ACE inhibitors are the first-line drugs for the treatment of hypertension in patients with diabetes mellitus, as they not only lower blood pressure but also have a renoprotective effect by reducing proteinuria and slowing the progression of diabetic nephropathy.  

β-blockers, calcium channel antagonists, and thiazide diuretics are also effective antihypertensive drugs, but they may have less renoprotective effects than ACE inhibitors and may even worsen proteinuria in some cases.   α-blockers are not recommended as first-line drugs for the treatment of hypertension, as they may cause orthostatic hypotension and other side effects.  

Therefore, based on the patient’s symptoms, history, and objective findings, the most appropriate drug for the treatment of hypertension is an ACE inhibitor, and the patient should be closely monitored for blood pressure control and renal function. The dose of the ACE inhibitor may need to be adjusted based on the patient’s response and side effects. In addition, the patient should be advised to follow a healthy lifestyle, such as reducing salt intake, exercising regularly, and avoiding smoking and excessive alcohol consumption, to further lower her blood pressure and prevent cardiovascular complications.


91. A 25 year old patient had pharyngitis 2 weeks ago. Now he complains about body temperature rise up to 38oC, general weakness, dyspnea during walking, swelling and shifting pain in the articulations. Objectively: cyanosis of lips, rhythmic pulse of poor volume – 100 bpm. Left cardiac border deviates outwards from the mediaclavicular line by 1 cm. The first heart sound is weakened on the apex, auscultation revealed systolic souffle. What is the most probable aetiological factor that caused this pathological process?

A. β-haemolytic streptococcus

B. Staphylococcus

C. Pneumococcus

D. Virus

E. Fungi


Answer: β-haemolytic streptococcus

Explanation

The patient’s symptoms of fever, weakness, dyspnea, joint pain, and cardiac findings of systolic murmur, weakened first heart sound, and left cardiac border deviation suggest a diagnosis of acute rheumatic fever (ARF), which is a multisystem inflammatory disease that can occur after a group A β-hemolytic streptococcal (GAS) infection, such as pharyngitis.  

ARF is an autoimmune response to the GAS infection, in which the antibodies produced against the GAS bacteria cross-react with the patient’s own tissues, leading to inflammation and damage to the heart, joints, and other organs.   Staphylococcus, pneumococcus, virus, and fungi can also cause infections, but they are less likely to cause ARF, as they do not have the same antigenic similarity to the patient’s own tissues as GAS.  

Therefore, based on the patient’s symptoms and history of pharyngitis, the most probable etiological factor that caused this pathological process is β-hemolytic streptococcus, and the patient should be treated with antibiotics, such as penicillin or erythromycin, to eradicate the GAS infection and prevent further complications of ARF, such as rheumatic heart disease. In addition, the patient should be closely monitored for cardiac function and joint involvement, and appropriate treatment, such as anti-inflammatory drugs and bed rest, should be provided as needed.

Early diagnosis and treatment of ARF can prevent long-term sequelae and improve the patient’s prognosis.


92. A 50 year old locksmith was diagnosed with typhoid fever. The patient lives in a separate apartment with all facilities. Apart of him there are also 2 adults in his family. What actions should be taken about persons communicating with the patient?

A. Bacteriological study

B. Antibiotic prophylaxis

C. Isolation

D. Dispensary observation

E. Vaccination


Answer: Bacteriological study

Explanation

Typhoid fever is a highly contagious bacterial infection caused by Salmonella typhi, which is transmitted through contaminated food, water, or fecal-oral route. Therefore, it is important to identify and isolate the source of the infection and take appropriate measures to prevent its spread to others.  

In this case, the patient lives in a separate apartment with his family, but it is still possible that they may have been exposed to the infection through contact with the patient or contaminated objects in the apartment.   Bacteriological study, which involves testing the patient’s blood, stool, or urine for the presence of S. typhi, can confirm the diagnosis of typhoid fever and help identify the source of the infection. If the bacteriological study is positive, the patient and his family members should be treated with antibiotics to eradicate the infection and prevent its further spread.  

Antibiotic prophylaxis, which involves giving antibiotics to people who have been exposed to typhoid fever to prevent infection, may be considered for high-risk individuals, such as family members who have had close contact with the patient, but it should not be used as a substitute for proper diagnosis and treatment of the infection.  

Isolation, which involves separating the patient from healthy individuals to prevent the spread of the infection, may be necessary in severe cases of typhoid fever or outbreaks in community settings, but it is not usually required for individual cases in a home setting.  

Dispensary observation, which involves monitoring the patient and his contacts for symptoms of typhoid fever and providing appropriate treatment if necessary, may be recommended to ensure early detection and management of the infection.   Vaccination, which involves giving a vaccine against typhoid fever to prevent infection, may be recommended for individuals at high risk of exposure, such as travelers to endemic areas, but it is not usually indicated for household contacts of a confirmed case of typhoid fever.  

Therefore, based on the patient’s diagnosis of typhoid fever and living situation, the most appropriate action to take about persons communicating with the patient is bacteriological study to confirm the diagnosis and identify the source of the infection, followed by appropriate treatment and monitoring as needed.


93. A 39 year old patient complained about morning headache, appetite loss, nausea, morning vomiting, periodic nasal haemorrhages. The patient had acute glomerulonephritis at the age of 15. Examination revealed rise of arterial pressure up to 220/130 mm Hg, skin haemorrhages on his arms and legs, pallor of skin and mucous membranes. What biochemical index has the greatest diagnostic importance in this case?

A. Blood creatinine

B. Blood bilirubin

C. Blood sodium

D. Uric acid

E. Fibrinogen


Answer:  Blood creatinine 

Explanation

The patient’s symptoms of morning headache, appetite loss, nausea, vomiting, nasal hemorrhages, high blood pressure, skin hemorrhages, and pallor suggest a diagnosis of hypertensive crisis, which can be caused by various underlying conditions, such as renal disease, autoimmune disorders, or vascular abnormalities.  

The patient’s history of acute glomerulonephritis at the age of 15 raises the possibility of chronic kidney disease, which can lead to hypertension and other systemic complications, such as anemia and bleeding disorders.   Therefore, the most important biochemical index in this case is blood creatinine, which is a marker of renal function and can help identify the degree of kidney damage and the need for urgent intervention.  


High levels of blood creatinine indicate impaired kidney function, which can lead to fluid and electrolyte imbalances, hypertension, and other complications. In addition, blood urea nitrogen (BUN) and estimated glomerular filtration rate (eGFR) may also be useful indicators of renal function and may help determine the cause and severity of the hypertensive crisis.  

Blood bilirubin, blood sodium, uric acid, and fibrinogen may be abnormal in hypertensive crisis, but they are not specific or sensitive enough to diagnose or monitor the underlying renal dysfunction.  

Therefore, based on the patient’s symptoms, history, and objective findings, the most important biochemical index for the diagnosis and management of this case is blood creatinine, and the patient should be referred to a specialist in nephrology or hypertension for further evaluation and treatment. Early detection and management of renal dysfunction and hypertension can prevent further complications and improve the patient’s prognosis.


94. A 43 year old patient was admitted to the infectious diseases hospital with high body temperature and intense headache. The iIlness has lasted for 2 days. Examination revealed a carbuncle on his forearm. The area around it was apparently edematic and slightly painful. Regional lymphadenitis and hepatolienal syndrome were also present. It is known from the anamnesis that the patient works at a cattle-breeding farm. What disease should be suspected in the first place?

A. Anthrax

B. Erysipelas

C. Erysipeloid

D. Skin cancer

E. Eczema


Answer: Anthrax

Explanation

The patient’s symptoms of high fever, intense headache, carbuncle on the forearm, regional lymphadenitis, and hepatosplenomegaly suggest a diagnosis of anthrax, which is a bacterial infection caused by Bacillus anthracis, a spore-forming organism found in soil and animal products.  

Anthrax can occur in three forms: cutaneous, inhalational, and gastrointestinal. Cutaneous anthrax is the most common form and is characterized by a painless, pruritic papule that progresses to a vesicle, then a painless ulcer with a characteristic black eschar in the center.  

The patient’s occupation at a cattle-breeding farm raises the possibility of exposure to spores from infected animals or contaminated soil or products, which can lead to cutaneous anthrax.   Erysipelas, erysipeloid, skin cancer, and eczema may also cause skin lesions and systemic symptoms, but they are less likely to present with the combination of symptoms and occupational history seen in this case.  

Therefore, based on the patient’s symptoms, history, and objective findings, the most likely diagnosis is cutaneous anthrax, and the patient should be treated with antibiotics, such as ciprofloxacin or doxycycline, to eradicate the infection and prevent further complications. In addition, the patient should be closely monitored for signs of systemic involvement, such as sepsis or meningitis, and appropriate supportive care, such as fluid and electrolyte management, should be provided as needed.

Furthermore, appropriate measures should be taken to prevent the spread of the infection, such as isolation and decontamination of the patient and his surroundings, and public health authorities should be notified of the suspected or confirmed case of anthrax.


95. Study of morbidity with temporary disability among workers of a machine building plant revealed that average duration of a case was 20 days. What diseases influenced upon the index value?

A. Chronic

B. Acute

C. Subacute

D. Preexisting diseases

E. Hard to determine


Answer:  Chronic

Explanation

The study of morbidity with temporary disability among workers of a machine building plant revealed an average duration of a case of 20 days. This suggests that the diseases that influenced the index value were likely chronic in nature, as acute and subacute conditions typically have shorter durations and may not result in prolonged disability.  

Chronic diseases are characterized by long-term persistence and may require ongoing management and treatment to prevent complications and disability. Examples of chronic diseases that may contribute to morbidity with temporary disability among workers of a machine building plant include musculoskeletal disorders, such as back pain or repetitive strain injuries, respiratory diseases, such as asthma or chronic obstructive pulmonary disease (COPD), and mental health conditions, such as depression or anxiety.  

Preexisting diseases, such as chronic conditions that were present before the worker’s employment at the machine building plant, may also contribute to morbidity with temporary disability, as these conditions may be exacerbated by the physical demands and occupational hazards of the job.  

Therefore, based on the information provided, the most likely answer is A. Chronic, as chronic diseases are more likely to result in prolonged disability and may be a significant contributor to the morbidity with temporary disability among workers of the machine building plant.


96. A 14 year old child suffers from vegetovascular dystonia of pubertal period. He has got sympathoadrenal atack. What medicine should be used for attack reduction?

A. Obsidan

B. No-shpa

C. Amysyl

D. Aminophylline

E. Corglicone


Answer: Obsidan

Explanation

Vegetovascular dystonia (VVD) is a functional disorder of the autonomic nervous system that can occur during puberty. Sympathoadrenal attacks, also known as panic attacks, are a common feature of VVD and can be characterized by symptoms such as palpitations, sweating, tremors, and anxiety.  

Obsidan, also known as propranolol, is a beta-blocker medication that can be used to reduce the symptoms of sympathoadrenal attacks by blocking the action of adrenaline and other stress hormones on the heart and blood vessels. Propranolol has been shown to be effective in reducing the frequency and severity of panic attacks and other symptoms of VVD.  

No-shpa, also known as drotaverine, is a spasmolytic medication that can be used to relieve abdominal pain, menstrual cramps, and other types of smooth muscle spasms, but it is not indicated for the treatment of sympathoadrenal attacks.   Amysyl, also known as amyl nitrite, is a vasodilator medication that can be used to relieve chest pain and other symptoms of angina, but it is not indicated for the treatment of sympathoadrenal attacks.  

Aminophylline is a bronchodilator medication that can be used to treat asthma and other respiratory conditions, but it is not indicated for the treatment of sympathoadrenal attacks.   Corglicone, also known as molsidomine, is a vasodilator medication that can be used to treat angina and other cardiovascular conditions, but it is not indicated for the treatment of sympathoadrenal attacks.  

Therefore, based on the patient’s diagnosis of VVD with sympathoadrenal attacks, the most appropriate medication to use for attack reduction is Obsidan (propranolol), under the supervision of a healthcare provider.


97. A 52 year old male patient complains about attacks of asphyxia, pain in his left side during respiration. These manifestations turned up all of a sudden. It is known from his anamnesis that he had been treated for thrombophlebitis of the right leg for the last month. In the admission ward the patient suddenly lost consciousness, there was a sudden attack of asphyxia and pain in his left side. Objectively: heart rate – 102/min, respiratory rate – 28/min, AP- 90/70 mm Hg. Auscultation revealed diastolic shock above the pulmonary artery, gallop rhythm, small bubbling rales above the lungs under the scapula on the right, pleural friction rub. What examination method will be the most informative for a diagnosis?

A. Angiography of pulmonary vessels

B. Echocardioscopy

C. Study of external respiration function

D. ECG E. Coagulogram


Answer: Angiography of pulmonary vessels

Explanation

The patient’s symptoms, including sudden onset of asphyxia and pain in the left side, along with the presence of diastolic shock above the pulmonary artery, gallop rhythm, and small bubbling rales above the lungs, suggest a potential pulmonary embolism. The fact that the patient had been treated for thrombophlebitis of the right leg further supports this suspicion, as deep vein thrombosis (DVT) is a known risk factor for pulmonary embolism.  

Angiography of the pulmonary vessels is a diagnostic test that involves using contrast dye and X-rays to visualize the blood vessels of the lungs. This test can confirm the presence of a pulmonary embolism by showing a blockage or narrowing in one or more pulmonary arteries.

It is considered the gold standard for diagnosing pulmonary embolism.   Echocardiography (Option B) can also be useful in diagnosing pulmonary embolism, as it can show signs of right heart strain or dysfunction, which can occur as a result of the increased pressure in the pulmonary artery caused by the embolism. However, angiography is generally considered more specific and sensitive for diagnosing pulmonary embolism.  

Studying the patient’s external respiration function (Option C) through tests such as spirometry or arterial blood gas analysis can provide information about the patient’s lung function, but it is not specific for diagnosing pulmonary embolism.   ECG (Option D) and coagulogram (Option E) can provide important information about the patient’s heart function and clotting status, respectively, but they are not specific for diagnosing pulmonary embolism.   It is important to note that any potential diagnostic tests should be considered in the context of the patient’s overall clinical presentation and medical history.


98. A 4 month old child fell seriously ill: body temperature rose up to 38, 5oC, the child became inert and had a single vomiting. 10 hours later there appeared rash over the buttocks and lower limbs in form of petechiae, spots and papules. Some haemorrhagic elements have necrosis in the centre. What is the most probable disease?

A. Meningococcemia

B. Rubella

C. Influenza

D. Haemorrhagic vasculitis

E. Scarlet fever


Answer:  Meningococcemia

Explanation

Meningococcemia is a serious bacterial infection caused by the bacteria Neisseria meningitidis. It can cause a range of symptoms, including fever, lethargy, vomiting, and a characteristic rash consisting of petechiae, spots, and papules that can progress to larger, hemorrhagic lesions with necrosis in the center. The rash typically appears first on the lower limbs and buttocks before spreading to other parts of the body.  

Given the sudden onset of high fever, lethargy, vomiting, and appearance of the characteristic rash with hemorrhagic elements, meningococcemia is the most likely diagnosis in this case. It is a medical emergency and requires urgent treatment with antibiotics and supportive measures, such as intravenous fluids, oxygen therapy, and close monitoring of vital signs.  

Rubella (Option B), influenza (Option C), and scarlet fever (Option E) can also cause fever and rash, but they typically have different clinical presentations and are less likely to cause the characteristic petechial and hemorrhagic rash seen in meningococcemia.  

Haemorrhagic vasculitis (Option D), also known as Henoch-Schonlein purpura, is a rare autoimmune disorder that can cause a purpuric rash, joint pain, abdominal pain, and kidney problems. However, it is less likely in this case given the sudden onset of symptoms and the presence of fever and vomiting, which are not typically seen in Henoch-Schonlein purpura.


99. A 13 year old girl was admitted to the cardiological department because of pain in the muscles and joints. Examination of her face revealed an edematic erythema in form of butterfly in the region of nose bridge and cheeks. What is the most probable diagnosis?

A. Systemic lupus erythematosus

B. Rheumatism

C. Dermatomyositis

D. Rheumatoid arthritis

E. Periarteritis nodosa


Answer: Systemic lupus erythematosus

Explanation

Systemic lupus erythematosus (SLE) is an autoimmune disease that can affect various organs and tissues in the body, including the skin, joints, kidneys, and heart. One of the hallmark features of SLE is a butterfly-shaped rash that appears on the face, particularly on the nose bridge and cheeks.

This rash is called malar rash or butterfly rash and is often associated with edema and erythema.   In addition to the malar rash, patients with SLE can also experience joint pain and muscle weakness (as seen in this case), as well as other symptoms such as fever, fatigue, and sensitivity to sunlight. Diagnosis of SLE is based on a combination of clinical features, laboratory tests, and imaging studies.  

Rheumatism (Option B) is a general term used to describe a variety of rheumatic diseases that affect the joints, muscles, and bones. It is not a specific diagnosis and does not provide enough information to identify the underlying cause of the patient’s symptoms.   D

ermatomyositis (Option C) is another autoimmune disease that can cause muscle weakness and skin rash, but the rash in dermatomyositis tends to be different from the malar rash seen in SLE and is often located on other parts of the body, such as the knuckles, elbows, or knees.   Rheumatoid arthritis (Option D) is another autoimmune disease that primarily affects the joints, causing pain, swelling, and stiffness. It does not typically cause skin rash or muscle weakness as seen in this case.  

Periarteritis nodosa (Option E), also known as polyarteritis nodosa, is a rare type of vasculitis that affects the medium-sized arteries, leading to inflammation, damage, and narrowing of the blood vessels. It can cause a range of symptoms, including fever, muscle pain, joint pain, and skin rash, but it is less likely in this case given the presence of the malar rash and the absence of other characteristic features of polyarteritis nodosa.


100. A 15 year old girl suddenly got arthralgia, headache, nausea, vomiting; pain and muscle tension in the lumbar area; body temperature rose up to 38 − 39oC. Pasternatsky’s symptom was distinctly positive on the right. In the urine: bacteriuria, pyuria. What is the most probable diagnosis?

A. Acute pyelonephritis

B. Renal colic

C. Acute glomerulonephritis

D. Pararenal abscess

E. Cystitis


Answer: Acute pyelonephritis

Explanation

Acute pyelonephritis is a bacterial infection of the kidneys that can cause a range of symptoms, including fever, chills, arthralgia (joint pain), headache, nausea, vomiting, and pain in the lumbar area. The presence of bacteriuria and pyuria in the urine indicates a urinary tract infection (UTI) involving the kidneys.  

The positive Pasternatsky’s sign on the right side (tenderness over the right kidney with percussion) is a clinical sign that suggests inflammation or infection of the kidney, which is consistent with the diagnosis of acute pyelonephritis.  

Renal colic (Option B) is typically caused by kidney stones and can cause severe pain in the back or flank, but it is not typically associated with fever, nausea, vomiting, or positive Pasternatsky’s sign.   Acute glomerulonephritis (Option C) is a type of kidney inflammation that can cause hematuria (blood in the urine), proteinuria (protein in the urine), and hypertension, but it is less likely in this case given the presence of pyuria and positive Pasternatsky’s sign.  

Pararenal abscess (Option D) is a rare but serious complication of kidney infection that can cause fever, chills, and back pain, but it typically presents with other signs of infection, such as elevated white blood cell count and sepsis.   Cystitis (Option E) is a bacterial infection of the bladder that can cause dysuria (painful urination), frequency, and urgency, but it is less likely in this case given the presence of pyuria, which suggests involvement of the upper urinary tract (i.e., kidneys).
Join the conversation
0% Complete